Những câu hỏi liên quan
em ơi
Xem chi tiết
Nguyễn Việt Lâm
27 tháng 12 2020 lúc 11:15

ĐKXĐ: \(abc\ne0\)

\(a^3+b^3+3ab\left(a+b\right)+c^3-3ab\left(a+b\right)-3abc=0\)

\(\Leftrightarrow\left(a+b\right)^3+c^3-3ab\left(a+b+c\right)=0\)

\(\Leftrightarrow\left(a+b+c\right)\left(a^2+b^2+c^2+2ab-ac-bc\right)-3ab\left(a+b+c\right)=0\)

\(\Leftrightarrow\left(a+b+c\right)\left[\left(a-b\right)^2+\left(b-c\right)^2+\left(c-a\right)^2\right]=0\)

\(\Leftrightarrow\left[{}\begin{matrix}a+b+c=0\\a=b=c\end{matrix}\right.\)

TH1: \(a+b+c=0\)

\(P=\dfrac{\left(a+b\right)\left(b+c\right)\left(c+a\right)}{abc}=\dfrac{\left(-c\right)\left(-a\right)\left(-b\right)}{abc}=-1\)

TH2: \(a=b=c\Rightarrow P=\left(1+1\right)\left(1+1\right)\left(1+1\right)=8\)

Bình luận (0)
Big City Boy
Xem chi tiết
anonymous
20 tháng 12 2020 lúc 19:13

Xét 2 TH sau:

TH1: a+b+c=0

Khi đó:

\(M=\left(1+\dfrac{a}{b}\right)\left(1+\dfrac{b}{c}\right)\left(1+\dfrac{c}{a}\right)\\ =\dfrac{a+b}{b}.\dfrac{b+c}{c}.\dfrac{c+a}{a}\\ =\dfrac{-c}{b}.\dfrac{-a}{c}.\dfrac{-b}{a}\\ =-1\)

TH2: a+b+c khác 0

Ta có:

\(\dfrac{a+b}{c}=\dfrac{b+c}{a}=\dfrac{c+a}{b}=\dfrac{2\left(a+b+c\right)}{a+b+c}=2\)

Suy ra: a+b=2c; b+c=2a; c+a=2b

Do đó:

\(M=\left(1+\dfrac{a}{b}\right)\left(1+\dfrac{b}{c}\right)\left(1+\dfrac{c}{a}\right)\\ =\dfrac{a+b}{b}.\dfrac{b+c}{c}.\dfrac{c+a}{a}\\ =\dfrac{2c}{b}.\dfrac{2a}{c}.\dfrac{2b}{a}\\ =8\)

Bình luận (0)
anonymous
20 tháng 12 2020 lúc 19:13

Xét 2 TH sau:

TH1: a+b+c=0

Khi đó:

\(M=\left(1+\dfrac{a}{b}\right)\left(1+\dfrac{b}{c}\right)\left(1+\dfrac{c}{a}\right)\\ =\dfrac{a+b}{b}.\dfrac{b+c}{c}.\dfrac{c+a}{a}\\ =\dfrac{-c}{b}.\dfrac{-a}{c}.\dfrac{-b}{a}\\ =-1\)

TH2: a+b+c khác 0

Ta có:

\(\dfrac{a+b}{c}=\dfrac{b+c}{a}=\dfrac{c+a}{b}=\dfrac{2\left(a+b+c\right)}{a+b+c}=2\)

Suy ra: a+b=2c; b+c=2a; c+a=2b

Do đó:

\(M=\left(1+\dfrac{a}{b}\right)\left(1+\dfrac{b}{c}\right)\left(1+\dfrac{c}{a}\right)\\ =\dfrac{a+b}{b}.\dfrac{b+c}{c}.\dfrac{c+a}{a}\\ =\dfrac{2c}{b}.\dfrac{2a}{c}.\dfrac{2b}{a}\\ =8\)

Bình luận (0)
xKraken
10 tháng 2 2021 lúc 22:32

Bổ sung cho bạn Lương Thị Quỳnh Trang

Đặt \(\dfrac{a+b}{c}=\dfrac{b+c}{a}=\dfrac{c+a}{b}=k\left(k\in R\right)\)

\(\Rightarrow\left\{{}\begin{matrix}a+b=ck\\b+c=ak\\c+a=bk\end{matrix}\right.\)

Cộng 3 đẳng thức trên, ta có:

2(a + b + c) = (a + b + c)k

<=> (a + b + c)(k - 2) = 0

\(\Rightarrow\left[{}\begin{matrix}a+b+c=0\\k=2\end{matrix}\right.\)

Với a + b + c = 0 thì giải như bạn ở dưới

Với k = 2 \(\Rightarrow\left\{{}\begin{matrix}a+b=2c\\b+c=2a\\c+a=2b\end{matrix}\right.\Leftrightarrow\left\{{}\begin{matrix}a+b+c=3c\\a+b+c=3a\\a+b+c=3b\end{matrix}\right.\)

=> 3a = 3b = 3c (= a + b + c) <=> a = b = c 

\(M=\left(1+\dfrac{a}{b}\right)\left(1+\dfrac{b}{c}\right)\left(1+\dfrac{c}{a}\right)=2.2.2=8\)

Vậy M = 8

Bình luận (0)
Đặng Thị Trà My
Xem chi tiết
Mei Shine
7 tháng 12 2023 lúc 21:35

Ta có: \(\dfrac{a+b}{c}=\dfrac{b+c}{a}=\dfrac{c+a}{b}\)\(=\dfrac{2\left(a+b+c\right)}{a+b+c}=2\)

=> a+b=2c; b+c=2a; c+a=2b

Thay vào A ta được: A=((a+b)/b)((c+b)/c)((a+c)/a)

=2c/b.2a/c.2b/a=2.2.2=8

Bình luận (0)
Xem chi tiết
Nguyễn Thị Hằng Nga
Xem chi tiết
Chira Nguyên
Xem chi tiết
Nguyễn Lê Phước Thịnh
28 tháng 2 2021 lúc 10:55

Ta có: \(a^3+b^3+c^3=3abc\)

\(\Leftrightarrow a^3+b^3+c^3-3abc=0\)

\(\Leftrightarrow\left(a+b\right)^3+c^3-3ab\left(a+b\right)-3abc=0\)

\(\Leftrightarrow\left(a+b+c\right)\left[\left(a+b\right)^2-c\left(a+b\right)+c^2\right]-3ab\left(a+b+c\right)=0\)

\(\Leftrightarrow\left(a+b+c\right)\left[a^2+2ab+b^2-ac-bc+c^2-3ab\right]=0\)

\(\Leftrightarrow\left(a+b+c\right)\left(a^2+b^2+c^2-ab-bc-ac\right)=0\)

\(\Leftrightarrow\left(a+b+c\right)\cdot\left(2a^2+2b^2+2c^2-2ab-2bc-2ac\right)=0\)

\(\Leftrightarrow\left(a+b+c\right)\left[\left(a^2-2ab+b^2\right)+\left(b^2-2bc+c^2\right)+\left(a^2-2ac+c^2\right)\right]=0\)

\(\Leftrightarrow\left(a+b+c\right)\left[\left(a-b\right)^2+\left(b-c\right)^2+\left(a-c\right)^2\right]=0\)

\(\Leftrightarrow\left[{}\begin{matrix}a+b+c=0\\\left(a-b\right)^2+\left(b-c\right)^2+\left(a-c\right)^2=0\end{matrix}\right.\)

\(\Leftrightarrow\left[{}\begin{matrix}a+b+c=0\\a=b=c\end{matrix}\right.\)

Ta có: \(N=\left(1+\dfrac{a}{b}\right)\left(1+\dfrac{b}{c}\right)\left(1+\dfrac{c}{a}\right)\)

\(=\dfrac{a+b}{b}\cdot\dfrac{b+c}{c}\cdot\dfrac{a+c}{a}\)

Trường hợp 1: a+b+c=0

\(\Leftrightarrow\left\{{}\begin{matrix}a+b=-c\\a+c=-b\\b+c=-a\end{matrix}\right.\)

\(\Leftrightarrow N=\dfrac{-c}{b}\cdot\dfrac{-a}{c}\cdot\dfrac{-b}{a}=\dfrac{-\left(a\cdot b\cdot c\right)}{a\cdot b\cdot c}=-1\)

Trường hợp 2: a=b=c

\(\Leftrightarrow N=\dfrac{b+b}{b}\cdot\dfrac{a+a}{a}\cdot\dfrac{c+c}{c}=2\cdot2\cdot2=8\)

Bình luận (0)
OP︵JACK-FF
28 tháng 2 2021 lúc 10:59

1, Ta có a^3+b^3+c^3=3abc

-> a^3+b^3+c^3+3a^2b+3ab^2=3abc+3a^2b+3ab^2

-> (a+b)3 + c^3 - 3ab(a+b+c)=0

-> (a+b+c). ((a+b)^2-(a+b).c+c^2)-3ab(a+b+c)=0

-> (a+b+c)(a^2+2ab+b^2-ac-bc+c^2-3ab)=0

Th1: a+b+c=0

->P= a+b/2 . b+c/2 . c+a/2

= (-c)(-a)(-b)/2=-1

TH2 a^2+b^2+c^2-ab-bc-ca=0

->2a^2+2b^2+2c^2-2ab-abc-2ac=0

->(a^2-2ab+b^2)+(a^2-2ac+c^2)+(b^2-2bc+c^2)=0

-> (a-b)^2+(a-c)^2+(b-c)^2=0

Mà (a-b)^2+(a-c)^2+(b-c)^2>= 0

Dấu = xảy ra (=)a-b=0

                         b-c=0

                          a-c=0

-> a=b=c

->P= 1+a/b+1+b/c+1+c/a=2+2+2= 8

Bình luận (0)
Big City Boy
Xem chi tiết
Thịnh Gia Vân
19 tháng 12 2020 lúc 20:34

Bài này mình làm một lần ở trường rồi nhưng không có điện thoại chụp được:((

Ta có: \(\dfrac{a^3}{\left(a-b\right)\left(a-c\right)}+\dfrac{b^3}{\left(b-a\right)\left(b-c\right)}+\dfrac{c^3}{\left(c-a\right)\left(c-b\right)}=\dfrac{a^3\left(c-b\right)+b^3\left(a-c\right)-c^3\left(a-b\right)}{\left(a-b\right)\left(a-c\right)\left(c-b\right)}\)\(=\dfrac{a^3\left(c-b\right)+b^3a-b^3c-c^3a+c^3b}{\left(a-b\right)\left(a-c\right)\left(c-b\right)}=\dfrac{a^3\left(c-b\right)-a\left(c^3-b^3\right)+bc\left(c^2-b^2\right)}{\left(a-b\right)\left(a-c\right)\left(c-b\right)}=\dfrac{a^3\left(c-b\right)-a\left(c-b\right)\left(a^2+bc+b^2\right)+bc\left(c-b\right)\left(c+b\right)}{\left(a-b\right)\left(a-c\right)\left(c-b\right)}\)\(=\dfrac{\left(c-b\right)\left(a^3-ac^2-abc-ab^2+bc^2+b^2c\right)}{\left(a-b\right)\left(c-b\right)\left(a-c\right)}=\dfrac{\left(c-b\right)\left[a\left(a^2-b^2\right)-c^2\left(a-b\right)-bc\left(a-b\right)\right]}{\left(a-b\right)\left(a-c\right)\left(c-b\right)}\)\(=\dfrac{\left(c-b\right)\left[a\left(a-b\right)\left(a+b\right)-c\left(a-b\right)-bc\left(a-b\right)\right]}{\left(a-b\right)\left(a-c\right)\left(c-b\right)}=\dfrac{\left(c-b\right)\left(a-b\right)\left(a^2+ab-c-bc\right)}{\left(a-b\right)\left(a-c\right)\left(c-b\right)}\)

\(\dfrac{\left(c-b\right)\left(a-b\right)\left[a^2-c^2+ab-bc\right]}{\left(a-b\right)\left(a-c\right)\left(c-b\right)}=\dfrac{\left(c-b\right)\left(a-b\right)\left[\left(a-c\right)\left(a+c\right)+b\left(a-c\right)\right]}{\left(a-b\right)\left(a-c\right)\left(c-b\right)}=\dfrac{\left(c-b\right)\left(a-b\right)\left(a-c\right)\left(a+b+c\right)}{\left(a-b\right)\left(c-b\right)\left(a-c\right)}\)\(=a+b+c\)

Vì a, b, c là các số nguyên

=> a+b+c là các số nguyên

=> Đpcm.

Đấy mình làm chi tiết tiền tiệt lắm luôn, không hiểu thì mình chịu rồi, trời lạnh mà đánh máy nhiều thế này buốt tay lắm luôn:vv

Bình luận (0)
Phạm Kim Oanh
Xem chi tiết
Bùi Đức Huy Hoàng
17 tháng 3 2022 lúc 18:19

a) phương trình \(x^3-3x^2+1\) có 3 nghiệm thực phân biệt là a,b,c(đề bài). Áp dụng Định lí Vi-ét cho đa thức bậc 3 ta có:\(\left\{{}\begin{matrix}a+b+c=3\\ab+bc+ac=0\\a.b.c=-1\end{matrix}\right.\)

ta có

      a+b+c=3

<=>\(\left(a+b+c\right)^2=9\)

<=>\(a^2+b^2+c^2+2ab+2bc+2ac=9\)

<=>\(a^2+b^2+c^2=9\)

<=>\(\left(a^2+b^2+c^2\right)^2=81\)

<=>\(a^4+b^4+c^4+2\left(a^2b^2+b^2c^2+a^2c^2\right)=81\)(1)

ta có ab+bc+ac=0

   <=>\(\left(ab+bc+ac\right)^2=0\)

   <=>\(a^2b^2+b^2c^2+a^2c^2+2abc\left(a+b+c\right)=0\)

   <=>\(a^2b^2+b^2c^2+a^2c^2-2.1.3=0\)

   <=>\(a^2b^2+b^2c^2+a^2c^2=6\)(2)

Thay (2) vào (1) ta có \(a^4+b^4+c^4+2.6=81\)

                                <=>\(a^4+b^4+c^4=69\)

Bình luận (0)
Bùi Đức Huy Hoàng
17 tháng 3 2022 lúc 19:11

b) \(\dfrac{a+1}{\left(b+c\right)\left(1-a\right)+1}=\dfrac{a+1}{\left(3-a\right)\left(1-a\right)+1}=\dfrac{a+1}{3+a^2-4a+1}=\dfrac{a+1}{a^2-4a+4}=\dfrac{a+1}{\left(a-2\right)^2}\)

cmtt =>\(B=\dfrac{a+1}{\left(a-2\right)^2}+\dfrac{b+1}{\left(b-2\right)^2}+\dfrac{c+1}{\left(c-2\right)^2}\)=\(\dfrac{1}{a-2}+\dfrac{1}{b-2}+\dfrac{1}{c-2}+3\left[\dfrac{1}{\left(a-2\right)^2}+\dfrac{1}{\left(b-2\right)^2}+\dfrac{1}{\left(c-2\right)^2}\right]\)=\(\dfrac{3\left[\left(a-2\right)\left(b-2\right)\right]^2+3\left[\left(b-2\right)\left(c-a\right)\right]^2+3\left[\left(c-2\right)\left(a-2\right)\right]^2}{\left[\left(a-2\right)\left(b-2\right)\left(c-2\right)\right]^2}\)

đặt t=(a-2)(b-2);u=(b-2)(c-2);v=(c-2)(a-2)     =>t+u+v=0

B thành \(\dfrac{3\left(t^2+u^2+v^2\right)}{t.u.v}\) bạn biến đổi để xuất hiện t+u+v

=>B=\(\dfrac{3\left(t+u+v\right)^2-6\left(t.u+u.v+t.v\right)}{t.u.v}=\dfrac{-6.\left(a-2\right)\left(b-2\right)\left(c-2\right)\left(a-2+b-2+c-2\right)}{t.u.v}=\dfrac{18}{\left(a-2\right)\left(b-2\right)\left(c-2\right)}\)

(a-2)(b-2)(c-2)= abc-2(ab+bc+ac)+4(a+b+c)-8=12-9=3

Vậy B=3

Bình luận (2)
Bùi Đức Huy Hoàng
17 tháng 3 2022 lúc 19:28

c) ta có \(\dfrac{a^3}{a^2+2bc}=\dfrac{a^3}{a^2-2ac-2ab}=\dfrac{a^2}{a-2c-2b}=\dfrac{a^2}{3a-2\left(a+b+c\right)}=\dfrac{a^2}{3\left(a-2\right)}\)

cmtt =>C=\(\dfrac{a^2}{3\left(a-2\right)}+\dfrac{b^2}{3\left(b-2\right)}+\dfrac{c^2}{3\left(c-2\right)}=\dfrac{a^2\left(b-2\right)\left(c-2\right)+b^2\left(a-2\right)\left(c-2\right)+c^2\left(a-2\right)\left(b-2\right)}{3\left(a-2\right)\left(b-2\right)\left(c-2\right)}\)

bạn nhân vô thì ra C=\(\dfrac{4a^2-2a\left(ab+ac\right)-a+4b^2-2b\left(bc+ab\right)-b+4c^2-2c\left(ac+bc\right)-c}{3\left(a-2\right)\left(b-2\right)\left(c-2\right)}=\dfrac{ }{ }4\dfrac{ }{ }=\dfrac{4\left(a^2+b^2+c^2\right)-\left(a+b+c\right)+6abc}{3\left(a-2\right)\left(b-2\right)\left(c-2\right)}=\dfrac{4.9-3-6}{3.3}=\dfrac{27}{9}=3\)

Bình luận (0)
Vịtt Tên Hiền
Xem chi tiết
Hoang Hung Quan
9 tháng 4 2017 lúc 19:03

Giải:

Từ \(a^3+b^3+c^3=3abc\Leftrightarrow\)\(\left[{}\begin{matrix}a+b+c=0\\a=b=c\end{matrix}\right.\)

Ta xét các trường hợp:

Trường hợp \(1\): Nếu \(a+b+c=0\) thì:

\(\Rightarrow\left\{{}\begin{matrix}a+b=-c\\b+c=-a\\a+c=-b\end{matrix}\right.\)

Thay vào \(P\) ta có:

\(P=\left(1+\dfrac{a}{b}\right)\left(1+\dfrac{b}{c}\right)\left(1+\dfrac{c}{a}\right)\)

\(=\left(\dfrac{a+b}{b}\right)\left(\dfrac{b+c}{c}\right)\left(\dfrac{a+c}{c}\right)\)

\(=\dfrac{-c}{b}.\dfrac{-a}{c}.\dfrac{-b}{a}=\dfrac{\cdot\left(-c\right).\left(-a\right).\left(-b\right)}{b.c.a}=-1\)

Trường hợp \(2\): Nếu \(a=b=c\) thì:

\(P=\left(1+\dfrac{a}{b}\right)\left(1+\dfrac{b}{c}\right)\left(1+\dfrac{c}{a}\right)\)

\(=\left(1+\dfrac{a}{a}\right)\left(1+\dfrac{a}{a}\right)\left(1+\dfrac{a}{a}\right)\)

\(=\left(1+1\right)\left(1+1\right)\left(1+1\right)\)

\(=2.2.2=8\)

Vậy \(P=-1\) hoặc \(P=8\)

Bình luận (0)
Trần Ánh Ngọc
8 tháng 4 2017 lúc 18:18

ta có : a3+b3+c3-3abc=0

\(\Rightarrow\)(a+b)3+c3-3abc-3a2b-3ab2=0

\(\Rightarrow\)(a+b+c)(a2+b2+c2+2ab-ac-bc)-3ab(a+b+c)=0

\(\Rightarrow\)(a+b+c)(a2+b2+c2-ab-ac-bc)=0

\(\Rightarrow\)\(\left[{}\begin{matrix}a+b+c=0\\a^2+b^2+c^2-ab-bc-ac=0\end{matrix}\right.\)

\(\Rightarrow\)\(\left[{}\begin{matrix}\left\{{}\begin{matrix}a=-\left(b+c\right)\\b=-\left(a+c\right)\\c=-\left(a+b\right)\end{matrix}\right.\\\left(a+b+c\right)^2+a^2+b^2+c^2=0\Leftrightarrow a=b=c=0\left(bỏ\right)\end{matrix}\right.\)ta có P=(1+\(\dfrac{a}{b}\))(1+\(\dfrac{b}{c}\))(1+\(\dfrac{c}{a}\))

\(\Leftrightarrow\)p=\(\left(\dfrac{b+a}{b}\right)\left(\dfrac{c+b}{c}\right)\left(\dfrac{a+c}{a}\right)\)

\(\Leftrightarrow P=\left(\dfrac{-c}{b}\right)\left(\dfrac{-a}{c}\right)\left(\dfrac{-b}{a}\right)\)

\(\Leftrightarrow\)P=-1

Bình luận (2)
Lê Thành Vinh
8 tháng 4 2017 lúc 18:23

a3+b3+c3=3abc

<=>a3+b3+c3-3abc=0

Bình luận (0)